Evaluate $\sum_{n=1}^{\infty}\frac{H_{n}^{(2)}}{n^{4}}$

calculusclosed-formdefinite integralsharmonic-numbersintegration

It may be rather tedious and I will have to delve into deeper, but I have a little something. We probably already know this one. The thing is, the first one results in yet another Euler sum. But, I think we can get it without contours. It may require some work, though.

$$
\sum_{n=1}^\infty\frac{H_n^{(2)}}{n^{4}}
=
\sum_{n=1}^\infty\sum_{k=1}^n
\frac 1 {k^4}
=
\sum_{n=1}^\infty\frac 1{n^2}
\sum_{k=n}^\infty\frac 1{k^4}\ .
$$

This result:

$$
\sum_{n=1}^{\infty}\frac{H_n^{(2)}}{n^4}
=
\zeta(2)\zeta(4)+\zeta(6)
-\sum_{n=1}^{\infty}\frac{H_n^{(4)}}{n^2}
$$

where that sum on the end is equal to
$$
\sum_{n=1}^{\infty}\frac{H_n^{(4)}}{n^2}
=\frac{37}{12}\zeta(6)-\zeta^{2}(3)\ .
$$

Which matches up if we change it all to $\zeta(6)$

i.e. $\displaystyle \zeta(4)\zeta(2)=\frac{7}{4}\zeta(6)$

and
$$
\sum_{n=1}^{\infty}\frac{H_n^{(2)}}{n^4}
=\zeta(2)\zeta(4)
+ \int_0^1\frac{\log(x)\operatorname{Li}_4(x)}{1-x}\; dx
$$

My main question is how to evaluate the last integral. Any better ways to evaluate the main Euler sum are welcome.

Best Answer

\begin{align}J&=\int_0^1\frac{\text{Li}_4(x)\ln x}{1-x}dx\\ &=-\frac{1}{6}\int_0^1\int_0^1\frac{x\ln^3 t\ln x}{(1-x)(1-tx)}dtdx\\ &=-\frac{1}{6}\int_0^1\int_0^1\left(\frac{\ln^3 t\ln x}{(1-x)(1-t)}-\frac{\ln^3 t\ln x}{(1-tx)(1-t)}\right)dtdx\\ &=-\zeta(2)\zeta(4)+\frac{1}{6}\int_0^1\int_0^1\frac{\ln^3 t\Big(\ln(t x)-\ln t\Big)}{(1-tx)(1-t)}dtdx\\ &=-\zeta(2)\zeta(4)+\frac{1}{6}\int_0^1 \frac{\ln^3 t}{t(1-t)}\left(\int_0^t\frac{\ln x}{1-x}dx\right)dt-\frac{1}{6}\int_0^1 \frac{\ln^4 t}{t(1-t)}\left(\int_0^t\frac{1}{1-x}dx\right)dt\\ &\overset{\text{IBP}}=-\zeta(2)\zeta(4)-\frac{1}{24}\int_0^1\frac{\ln^5 t}{1-t}dt+\frac{1}{6}\int_0^1\frac{\ln^3 t}{1-t}\left(\int_0^t\frac{\ln x}{1-x}dx\right)dt+\frac{1}{30}\int_0^1\frac{\ln^5 t}{1-t}dt+\\ &\frac{1}{6}\int_0^1\frac{\ln^4 t\ln(1-t)}{1-t}dt\\ &=\zeta(6)-\zeta(2)\zeta(4)+\frac{1}{6}\underbrace{\int_0^1\frac{\ln^3 t}{1-t}\left(\int_0^t\frac{\ln x}{1-x}dx \right)dt}_{=A}+\frac{1}{6}\int_0^1\frac{\ln^4 t\ln(1-t)}{1-t}dt\\ A&=\int_0^1\int_0^1\frac{t\ln(tx)\ln^3 t}{(1-t)(1-tx)}dtdx=\int_0^1\int_0^1\left(\frac{\ln(tx)\ln^3 t}{(1-t)(1-x)}-\frac{\ln(tx)\ln^3 t}{(1-x)(1-tx)}\right)dtdx\\ &\overset{\text{Fubini}}=6\zeta(2)\zeta(4)+\int_0^1\int_0^1\left(\frac{\ln^4 t}{(1-t)(1-x)}-\frac{\ln(tx)\Big(\ln(tx)-\ln x\Big)^3}{(1-x)(1-tx)}\right)dtdx\\ &=6\zeta(2)\zeta(4)+\int_0^1\int_0^1\left(\frac{\ln^4 t}{(1-t)(1-x)}-\frac{\ln^4(tx)}{(1-x)(1-tx)}\right)dtdx+\\&3\!\int_0^1\!\!\int_0^1\frac{\ln^3(tx)\ln x}{(1-x)(1-tx)}\!dtdx-3\int_0^1\int_0^1\frac{\ln^2(tx)\ln^2 x}{(1-x)(1-tx)}dtdx+\\&\int_0^1\int_0^1\frac{\ln(tx)\ln^3 x}{(1-x)(1-tx)}dtdx\\ &=6\zeta(2)\zeta(4)+\int_0^1\frac{1}{1-x}\left(\int_0^1\frac{\ln^4 t}{1-t}dt-\frac{1}{x}\int_0^x\frac{\ln^4 t}{1-t}dt\right)dx+\\&3\int_0^1\frac{\ln x}{(1-x)x}\left(\int_0^x \frac{\ln^3 t}{1-t}dt\right)dx-3\int_0^1\frac{\ln^2 x}{(1-x)x}\left(\int_0^x \frac{\ln^2 t}{1-t}dt\right)dx+\\&\int_0^1\frac{\ln^3 x}{(1-x)x}\left(\int_0^x \frac{\ln t}{1-t}dt\right)dx\\ &\overset{\text{IBP}}=6\zeta(2)\zeta(4)-30\zeta(6)-\int_0^1\frac{\ln(1-t)\ln^4 t}{1-t}dt+3\underbrace{\int_0^1\frac{\ln x}{1-x}\left(\int_0^x \frac{\ln^3 t}{1-t}dt\right)dx}_{\text{IBP}}-\\ &3\underbrace{\int_0^1\frac{\ln^2 x}{1-x}\left(\int_0^x \frac{\ln^2 t}{1-t}dt\right)dx}_{=2\zeta(3)^2}+A\\ &=24\zeta(2)\zeta(4)-30\zeta(6)-6\zeta(3)^2-\int_0^1\frac{\ln(1-t)\ln^4 t}{1-t}dt-2A\\ &=8\zeta(2)\zeta(4)-10\zeta(6)-2\zeta(3)^2-\frac{1}{3}\int_0^1\frac{\ln(1-t)\ln^4 t}{1-t}dt\\ J&=-\frac{2}{3}\zeta(6)+\frac{1}{3}\zeta(2)\zeta(4)-\frac{1}{3}\zeta(3)^2+\frac{1}{9}\underbrace{\int_0^1\frac{\ln(1-t)\ln^4 t}{1-t}dt}_{=B} \end{align} \begin{align} B&\overset{\text{IBP}}=\left[\left(\int_0^t\frac{\ln^4 x}{1-x}dx-\int_0^1\frac{\ln^4 x}{1-x}dx\right)\ln(1-t)\right]_0^1+\\&\int_0^1\int_0^1\frac{1}{1-t}\left(\frac{t\ln^4(tx)}{1-tx}-\frac{\ln^4 x}{1-x}\right)dtdx\\ &=\int_0^1\int_0^1\left(\frac{\ln^4(tx)}{(1-t)(1-x)}-\frac{\ln^4(tx)}{(1-x)(1-tx)}-\frac{\ln^4 x}{(1-t)(1-x)}\right)dtdx\\ &=\int_0^1\int_0^1\frac{4\ln t\ln^3 x+6\ln^2 t\ln^2 x+4\ln^3 t\ln x}{(1-t)(1-x)}dtdx+\\ &\int_0^1\int_0^1\left(\frac{\ln^4 t}{(1-t)(1-x)}-\frac{\ln^4(tx)}{(1-x)(1-tx)}\right)dtdx\\ &\overset{\text{Fubini}}=24\zeta(2)\zeta(4)+24\zeta(3)^2+24\zeta(2)\zeta(4)-\int_0^1\frac{1}{x}\int_0^x\frac{\ln^4 t}{1-t}dt+\int_0^1\frac{1}{1-x}\int_x^1\frac{\ln^4 t}{1-t}dt\\ &\overset{\text{IBP}}=48\zeta(2)\zeta(4)+24\zeta(3)^2-120\zeta(6)-B\\ &=\boxed{24\zeta(2)\zeta(4)+12\zeta(3)^2-60\zeta(6)} \end{align} Therefore, \begin{align}\boxed{J=-\frac{22}{3}\zeta(6)+3\zeta(2)\zeta(4)+\zeta(3)^2=\zeta(3)^2-\frac{5\pi^6}{2268}}\end{align}